What kind of equipment would a giant-slayer use?












9












$begingroup$


My character is a giant-slayer living in the far North with his family. He is able to bend and generate magnetic fields. The only technology he could use would be something like Feudal Japan and/or Europe.



The giants are very slow creatures but 7 times stronger and bigger than humans. They can smell humans and feed on them. On the other hand the small amount of people in their territory determined their species to remain at 100 specimens.



Giving this information, what type of tools/weapons and even armor would he use?










share|improve this question











$endgroup$








  • 1




    $begingroup$
    Did you mean 18.5 times bigger? (Square cube law)
    $endgroup$
    – NofP
    Jan 8 at 8:12






  • 18




    $begingroup$
    5 smooth stones and a sling.
    $endgroup$
    – jpmc26
    Jan 8 at 8:40






  • 2




    $begingroup$
    historically this has always been a slingshot.
    $endgroup$
    – user6858980
    Jan 8 at 9:52






  • 3




    $begingroup$
    Do you mean seven times a human's weight, or seven times a human's height? The former gives you giants just under twice a human's height, the latter gives you giants that weigh 343x what a human does. And neither gives you a 7x strength factor, because muscle strength is proportional to the muscle's cross-sectional area.
    $endgroup$
    – Arcanist Lupus
    Jan 8 at 14:33






  • 1




    $begingroup$
    So it's giants vs Magneto?
    $endgroup$
    – Brian Risk
    Jan 8 at 18:30
















9












$begingroup$


My character is a giant-slayer living in the far North with his family. He is able to bend and generate magnetic fields. The only technology he could use would be something like Feudal Japan and/or Europe.



The giants are very slow creatures but 7 times stronger and bigger than humans. They can smell humans and feed on them. On the other hand the small amount of people in their territory determined their species to remain at 100 specimens.



Giving this information, what type of tools/weapons and even armor would he use?










share|improve this question











$endgroup$








  • 1




    $begingroup$
    Did you mean 18.5 times bigger? (Square cube law)
    $endgroup$
    – NofP
    Jan 8 at 8:12






  • 18




    $begingroup$
    5 smooth stones and a sling.
    $endgroup$
    – jpmc26
    Jan 8 at 8:40






  • 2




    $begingroup$
    historically this has always been a slingshot.
    $endgroup$
    – user6858980
    Jan 8 at 9:52






  • 3




    $begingroup$
    Do you mean seven times a human's weight, or seven times a human's height? The former gives you giants just under twice a human's height, the latter gives you giants that weigh 343x what a human does. And neither gives you a 7x strength factor, because muscle strength is proportional to the muscle's cross-sectional area.
    $endgroup$
    – Arcanist Lupus
    Jan 8 at 14:33






  • 1




    $begingroup$
    So it's giants vs Magneto?
    $endgroup$
    – Brian Risk
    Jan 8 at 18:30














9












9








9


1



$begingroup$


My character is a giant-slayer living in the far North with his family. He is able to bend and generate magnetic fields. The only technology he could use would be something like Feudal Japan and/or Europe.



The giants are very slow creatures but 7 times stronger and bigger than humans. They can smell humans and feed on them. On the other hand the small amount of people in their territory determined their species to remain at 100 specimens.



Giving this information, what type of tools/weapons and even armor would he use?










share|improve this question











$endgroup$




My character is a giant-slayer living in the far North with his family. He is able to bend and generate magnetic fields. The only technology he could use would be something like Feudal Japan and/or Europe.



The giants are very slow creatures but 7 times stronger and bigger than humans. They can smell humans and feed on them. On the other hand the small amount of people in their territory determined their species to remain at 100 specimens.



Giving this information, what type of tools/weapons and even armor would he use?







medieval armors tools






share|improve this question















share|improve this question













share|improve this question




share|improve this question








edited Jan 8 at 4:36









Monica Cellio

12.4k653116




12.4k653116










asked Jan 8 at 1:26









Edward ConstantinEdward Constantin

12418




12418








  • 1




    $begingroup$
    Did you mean 18.5 times bigger? (Square cube law)
    $endgroup$
    – NofP
    Jan 8 at 8:12






  • 18




    $begingroup$
    5 smooth stones and a sling.
    $endgroup$
    – jpmc26
    Jan 8 at 8:40






  • 2




    $begingroup$
    historically this has always been a slingshot.
    $endgroup$
    – user6858980
    Jan 8 at 9:52






  • 3




    $begingroup$
    Do you mean seven times a human's weight, or seven times a human's height? The former gives you giants just under twice a human's height, the latter gives you giants that weigh 343x what a human does. And neither gives you a 7x strength factor, because muscle strength is proportional to the muscle's cross-sectional area.
    $endgroup$
    – Arcanist Lupus
    Jan 8 at 14:33






  • 1




    $begingroup$
    So it's giants vs Magneto?
    $endgroup$
    – Brian Risk
    Jan 8 at 18:30














  • 1




    $begingroup$
    Did you mean 18.5 times bigger? (Square cube law)
    $endgroup$
    – NofP
    Jan 8 at 8:12






  • 18




    $begingroup$
    5 smooth stones and a sling.
    $endgroup$
    – jpmc26
    Jan 8 at 8:40






  • 2




    $begingroup$
    historically this has always been a slingshot.
    $endgroup$
    – user6858980
    Jan 8 at 9:52






  • 3




    $begingroup$
    Do you mean seven times a human's weight, or seven times a human's height? The former gives you giants just under twice a human's height, the latter gives you giants that weigh 343x what a human does. And neither gives you a 7x strength factor, because muscle strength is proportional to the muscle's cross-sectional area.
    $endgroup$
    – Arcanist Lupus
    Jan 8 at 14:33






  • 1




    $begingroup$
    So it's giants vs Magneto?
    $endgroup$
    – Brian Risk
    Jan 8 at 18:30








1




1




$begingroup$
Did you mean 18.5 times bigger? (Square cube law)
$endgroup$
– NofP
Jan 8 at 8:12




$begingroup$
Did you mean 18.5 times bigger? (Square cube law)
$endgroup$
– NofP
Jan 8 at 8:12




18




18




$begingroup$
5 smooth stones and a sling.
$endgroup$
– jpmc26
Jan 8 at 8:40




$begingroup$
5 smooth stones and a sling.
$endgroup$
– jpmc26
Jan 8 at 8:40




2




2




$begingroup$
historically this has always been a slingshot.
$endgroup$
– user6858980
Jan 8 at 9:52




$begingroup$
historically this has always been a slingshot.
$endgroup$
– user6858980
Jan 8 at 9:52




3




3




$begingroup$
Do you mean seven times a human's weight, or seven times a human's height? The former gives you giants just under twice a human's height, the latter gives you giants that weigh 343x what a human does. And neither gives you a 7x strength factor, because muscle strength is proportional to the muscle's cross-sectional area.
$endgroup$
– Arcanist Lupus
Jan 8 at 14:33




$begingroup$
Do you mean seven times a human's weight, or seven times a human's height? The former gives you giants just under twice a human's height, the latter gives you giants that weigh 343x what a human does. And neither gives you a 7x strength factor, because muscle strength is proportional to the muscle's cross-sectional area.
$endgroup$
– Arcanist Lupus
Jan 8 at 14:33




1




1




$begingroup$
So it's giants vs Magneto?
$endgroup$
– Brian Risk
Jan 8 at 18:30




$begingroup$
So it's giants vs Magneto?
$endgroup$
– Brian Risk
Jan 8 at 18:30










10 Answers
10






active

oldest

votes


















21












$begingroup$


  • Your first weapon is a shovel.


Drop an ant from an airplane and it lands safely on the ground (1), drop an elephant from 2 meters height and it breaks all its bones. Drop a giant... You simply dig traps, cover them up preferably in a way that people at people height can see warnings and a giant can't and wait. Even relatively shallow holes will break the arms and torso of the giant, if not his legs as well. This might not be a totally valid answer if you consider that the giant is surviving the square-cube law. As an alternative he could use a lot of thick ropes strung up around trees to trip the giant.




  • Make it bleed.


To pump all that blood around the giant is going to need a lot more blood pressure, and a cut on the leg for example is going to be much harder to stop bleeding for such a giant creature. Use bows, crossbows, traps or suddenly spring from the bush and use a large axe (not a sword, you need the force and cutting power of an axe, there's a reason we don't cut trees with swords), then try to hide in the brush again. The giant will take a while, but he's much more likely to bleed to death.




  • Viral warfare


Poisons might not be that readily available, a pit with multi-week old poop from different species as well as some dead critters will (2). Dip your weapons into that before you wound the giant. You could also put that stuff into food you put out in the forest, for example some dead humans to lure them in (criminals if it makes your conscience better, or if you don't have a conscience, use live ones).



(1) source as offered by Douwe: https://www.theverge.com/tldr/2016/6/15/11936802/ant-dropped-from-empire-state-building-science-experiment-mystery-solved



(2) https://en.m.wikipedia.org/wiki/History_of_biological_warfare






share|improve this answer











$endgroup$









  • 7




    $begingroup$
    Drop an ant from an airplane and it lands safely on the ground Accurate apparently, sorry for doubting you :)
    $endgroup$
    – Douwe
    Jan 8 at 12:17








  • 1




    $begingroup$
    The last one was an actual medieval tactic used to contaminate blades and arrows. To avoid dripping a (reusable) sword blade in the nasty stuff, they could also suspend it just above it for the night.
    $endgroup$
    – Eth
    Jan 8 at 12:58










  • $begingroup$
    @Douwe no problem. Many thanks for looking up sources and posting them. I'll add it to the answer.
    $endgroup$
    – Demigan
    Jan 8 at 14:20






  • 1




    $begingroup$
    "A rat is killed, a man is broken, a horse splashes." - JBS Haldane
    $endgroup$
    – Arcanist Lupus
    Jan 8 at 14:41






  • 1




    $begingroup$
    @Yakk No matter the pressure, the trap needs to be large enough for the foot to actually fall into. And then, the entire weight of the giant rests on the trap when it's supposed to give in. Thus, the supporting structures that are supposed to give in at about 343*70kg = 2.4t can easily discern this weight even from a few hundreds of kg. There is absolutely no need to design the trap to support 343 humans.
    $endgroup$
    – cmaster
    Jan 10 at 21:21



















9












$begingroup$

Since the giants are slow and strong, I would recommend just running around naked. That will make your hunter extremely light and nimble and so that they can recover from their I-Frame dodge rolls quickly. Armor is of little use if the creature your fighting is both 7 times larger and stronger. Nothing is going to really protect your from one of their attacks, so armor should really just be clothes based on the environment you are hunting in.



As for the weapon, I would recommend poison. There are many lethal poisons out there that even in small doses would kill a giant. So while a drop might kill several humans, a couple more drops will kill a giant. And since they are slow, you just stab them with a needle several times (the needles size will have to vary depending on the giants skin thickness and toughness) and run away and wait for the poison to do its work.






share|improve this answer









$endgroup$









  • 1




    $begingroup$
    Poisoned rapier, perhaps?
    $endgroup$
    – Arkenstein XII
    Jan 8 at 2:14






  • 1




    $begingroup$
    Well, you do have to consider how thick their skin is, small poisonous spiders can bite humans, but their fangs might not even penetrate our skin. Adult spiders, however are a different issue.
    $endgroup$
    – Mr.J
    Jan 8 at 3:21






  • 4




    $begingroup$
    a slingshot springs to mind.
    $endgroup$
    – Giu Piete
    Jan 8 at 5:42










  • $begingroup$
    /I would recommend just running around naked/. You answer every single question that way! Not saying you're wrong, of course.
    $endgroup$
    – Willk
    Jan 9 at 1:11












  • $begingroup$
    @Willk I've been playing dark souls lately... need to remove all my equipment for the quick roll. Gloves and boots are all the protection I need.
    $endgroup$
    – Shadowzee
    Jan 9 at 1:13



















7












$begingroup$

The smart path would be to use a crossbow or longbow and to shoot the giants with poisoned arrows while on horseback (He does not have to shoot on the move. He could just kite the giants by shooting and then moving away, then shooting again).



Keep in mind that the giants would probably be able to throw rocks at him, so he would have to maintain some distance. Also, the giants are quite large, so it would take a lot of poison to kill them. The smart thing to do would be for him to have several people accompany him.



He would want to avoid melee at any cost, as even a glancing blow from a giant could smash his head in, no matter what armor he wears. Digging pit traps with feces coated stakes on the bottom and luring giants into them could be viable. Carrying some caltrops and scattering them behind him might be useful, but the giant might just walk around those. They would be quite effective at night.



Also, he might want to wear at least a gambeson. Even if it would not protect against melee strikes or thrown rocks, smarter giants might throw clay pots full of small stones at him like hand grenades, and he would want some protection from that.






share|improve this answer









$endgroup$













  • $begingroup$
    If you don't have to kill the giant quickly, consider disease rather than poison - biological agents multiply, so a small dose will grow inside the giant until reaching lethal levels...
    $endgroup$
    – G0BLiN
    Jan 8 at 12:38






  • 1




    $begingroup$
    I totally forgot about the magnetic fields.
    $endgroup$
    – Richard Smith
    Jan 8 at 15:36



















3












$begingroup$

Fighting one or two giants are easy, but fighting a group? that's one thing to consider.



You can't become a "Giant slayer" just because you killed one, you have to have killed a plenty.



That being said, I do think giant killer is a wrong title as your giants are WAAAAAY to strong and WAAAAY to tall for someone to kill with a blade, even with poison. At that height, the most weakest point of the giant is the feet. big, range weapons will penetrate those but they are a burden to carry by one man.



A more plausible approach might be, The Giant killing army.



Your army can, scatter long, steal spike, drench them with poison, then cover them with forestry.



Your main weapon for killing them will be ballistas... Lots of them, as they are the weapon with the highest percentage of killing a giant without any casualties.



There should be one improvement however, you ballista must have a reloading mechanism similar to a Van Helsing's repeater crossbow because you need to reload as quickly as possible, but pack a punch too.



one of your problems however, is that you got 100 persons, give or take that some of them are women, or grandparents and kids, your army lacks manpower, 50 might hold for some time.



I also suggest to be on the defensive rather than taking the fight too them, setting up traps is the best way to kill the giants, you need all the manpower to protect your people and ensure continuity of the human race.



another problem is giant carcass, yeah you killed them, but if they die close to your encampment then then stench will kill you as well, so you better plan your traps, how you set your encampment, and how you can engage the giants with your ballistas on the encampment, and during the move.



you will have a pretty busy life if you ask me...






share|improve this answer









$endgroup$





















    3












    $begingroup$

    "He is able to bend and generate magnetic fields" - Does it mean he can manipulate metal and utilize it for combat? Well... in that case "Avatar: The Last Airbender" comes to mind with metal benders (rather less accurate, meaning it's more like throwing and shooting metallic objects), or X-Man with Magneto (full, precise control), or even Irelia (special weapon) in League of Legends.



    In both cases he'd carry large blades around, either swords or just the blade parts. When he fights he swings them around on range, pierces his opponents, performs attacks from their backs (when returning the blades). Against giants he would go for the weak spots, which he can reach unlike normal humans.



    For armor he'd either wear leather (in case the metal interferes with his abilities if it covers his body) or an armor with metallic parts, which he can optionally use as weapons or as defence. Alternatively his armor could consist of the weapons he is carrying around, meaning the blades are part of the armor (the shoulder pads or on the backside).






    share|improve this answer











    $endgroup$





















      2












      $begingroup$

      Rope. Think of the AT-AT's in Star Wars, just run around it and try to trip it. Other similar things would probably work also like nets. Once it is slowed down/downed, just stick it with the pointy end.



      If they are slow enough, maybe bring a large crossbow/ballista mounted on a carriage or something.



      Metallic spike traps, wall or ground mounted if they can manipulated by magnetic force, depending on strength of manipulation.
      enter image description here






      share|improve this answer











      $endgroup$













      • $begingroup$
        Metallic bolas might work better for a person capable of manipulating magnetic fields... :)
        $endgroup$
        – G0BLiN
        Jan 8 at 12:41



















      1












      $begingroup$

      Science



      If there are only 100 giants around at any time, then they are at the brink of extinction. The amount of inbreeding over generations is astounding. The giants are probably stupid to the extreme due to that. Also, with only 100 giants at any time they will hardly be able to keep a culture or be educated.



      Sinxe they hunt by smell, all the slayer needs to do is dressing scarecrows with clothes recently worn by actual people. Then fill the scarecrows with fresh but poisoned meat. Just wait for a few days before checking for each scarecrow.






      share|improve this answer









      $endgroup$













      • $begingroup$
        indeed they are stupid. They will be later controlled with the use of an artifact and this dude will fight them because the one controlling them want to get over a territory which includes his home place.
        $endgroup$
        – Edward Constantin
        Jan 8 at 11:12



















      1












      $begingroup$

      In the interest of using your hero's power over magnetic fields, let's try a couple of other suggestions.




      • Can he use the magnetic fields to accelerate a metal projectile beyond normal velocities? If you can take the average bow velocity of 300fps and get it up to 1200fps you could get away with something like "5 smooth ball-bearings and a sling".


      • Can you manipulate a steel object at a distance? For instance, could you plant a knife somewhere in the giant's chest at range, then use magnetism to work the knife into his vitals?


      • Now, to REALLY get rid of the giant, a magnetically confined Tokamak reactor...ok, off subject.


      • As for armor, a magnetically controlled shield would be very entertaining. For balance, you would probably need to make the effort to your hero comparable to the force applied to it. (A giant would still knock it around)







      share|improve this answer









      $endgroup$













      • $begingroup$
        How do you think the magnetic shield would work? Can you come up with a design?
        $endgroup$
        – Edward Constantin
        Jan 9 at 16:43






      • 1




        $begingroup$
        Hmm, my imagination only got as far as taking a normal steel shield and levitating it. If we want to be more scientific, we could consider magnetic repulsion. Take a shield and put a large magnet on the back side. That becomes more like a magnetic shock absorber. All the energy of the impact goes into your character, but over an extended period of time. Maybe your character can transfer some of that to the planet's magnetic field to not take the full impact?
        $endgroup$
        – UrQuan3
        Jan 10 at 18:33



















      1












      $begingroup$

      Your main character could use a rope or net studded or made of metal. The rope wraps around the giant to bring them down, and your main character can control the movement of the rope/net very precisely by manipulating the metal in it.
      Once bound, you have a number of ways to kill it; personally I would recommend just stabbing it until it bleeds out. Go for the eyes first, in case your character’s rope inexplicably fails.



      For armor, I agree with others; the strength of the armor doesn't matter when going against giants, so ease of movement is key. Clothing or leather is best.






      share|improve this answer









      $endgroup$





















        1












        $begingroup$

        How strong and precise magnetic field are we talking about here?



        It strong enough, he can give cannon balls so much kinetic energy (preferably from distance, for example in high levitating matal chair with seatbelts so no giant can reach him but he has nice view of battlefield) to massacre the poor giants to bloody equivalent of mashed potatoes.



        Even worse, if can generate ridiculously strong magnetic field (like magnetars) he would be God-like, being able to distort atoms themselves in Giant bodies (and elsewhere).






        share|improve this answer











        $endgroup$













        • $begingroup$
          He is not that strong. He can move heavy objects but not anything like creating magnetars
          $endgroup$
          – Edward Constantin
          Jan 9 at 16:42










        protected by L.Dutch Jan 8 at 10:24



        Thank you for your interest in this question.
        Because it has attracted low-quality or spam answers that had to be removed, posting an answer now requires 10 reputation on this site (the association bonus does not count).



        Would you like to answer one of these unanswered questions instead?














        10 Answers
        10






        active

        oldest

        votes








        10 Answers
        10






        active

        oldest

        votes









        active

        oldest

        votes






        active

        oldest

        votes









        21












        $begingroup$


        • Your first weapon is a shovel.


        Drop an ant from an airplane and it lands safely on the ground (1), drop an elephant from 2 meters height and it breaks all its bones. Drop a giant... You simply dig traps, cover them up preferably in a way that people at people height can see warnings and a giant can't and wait. Even relatively shallow holes will break the arms and torso of the giant, if not his legs as well. This might not be a totally valid answer if you consider that the giant is surviving the square-cube law. As an alternative he could use a lot of thick ropes strung up around trees to trip the giant.




        • Make it bleed.


        To pump all that blood around the giant is going to need a lot more blood pressure, and a cut on the leg for example is going to be much harder to stop bleeding for such a giant creature. Use bows, crossbows, traps or suddenly spring from the bush and use a large axe (not a sword, you need the force and cutting power of an axe, there's a reason we don't cut trees with swords), then try to hide in the brush again. The giant will take a while, but he's much more likely to bleed to death.




        • Viral warfare


        Poisons might not be that readily available, a pit with multi-week old poop from different species as well as some dead critters will (2). Dip your weapons into that before you wound the giant. You could also put that stuff into food you put out in the forest, for example some dead humans to lure them in (criminals if it makes your conscience better, or if you don't have a conscience, use live ones).



        (1) source as offered by Douwe: https://www.theverge.com/tldr/2016/6/15/11936802/ant-dropped-from-empire-state-building-science-experiment-mystery-solved



        (2) https://en.m.wikipedia.org/wiki/History_of_biological_warfare






        share|improve this answer











        $endgroup$









        • 7




          $begingroup$
          Drop an ant from an airplane and it lands safely on the ground Accurate apparently, sorry for doubting you :)
          $endgroup$
          – Douwe
          Jan 8 at 12:17








        • 1




          $begingroup$
          The last one was an actual medieval tactic used to contaminate blades and arrows. To avoid dripping a (reusable) sword blade in the nasty stuff, they could also suspend it just above it for the night.
          $endgroup$
          – Eth
          Jan 8 at 12:58










        • $begingroup$
          @Douwe no problem. Many thanks for looking up sources and posting them. I'll add it to the answer.
          $endgroup$
          – Demigan
          Jan 8 at 14:20






        • 1




          $begingroup$
          "A rat is killed, a man is broken, a horse splashes." - JBS Haldane
          $endgroup$
          – Arcanist Lupus
          Jan 8 at 14:41






        • 1




          $begingroup$
          @Yakk No matter the pressure, the trap needs to be large enough for the foot to actually fall into. And then, the entire weight of the giant rests on the trap when it's supposed to give in. Thus, the supporting structures that are supposed to give in at about 343*70kg = 2.4t can easily discern this weight even from a few hundreds of kg. There is absolutely no need to design the trap to support 343 humans.
          $endgroup$
          – cmaster
          Jan 10 at 21:21
















        21












        $begingroup$


        • Your first weapon is a shovel.


        Drop an ant from an airplane and it lands safely on the ground (1), drop an elephant from 2 meters height and it breaks all its bones. Drop a giant... You simply dig traps, cover them up preferably in a way that people at people height can see warnings and a giant can't and wait. Even relatively shallow holes will break the arms and torso of the giant, if not his legs as well. This might not be a totally valid answer if you consider that the giant is surviving the square-cube law. As an alternative he could use a lot of thick ropes strung up around trees to trip the giant.




        • Make it bleed.


        To pump all that blood around the giant is going to need a lot more blood pressure, and a cut on the leg for example is going to be much harder to stop bleeding for such a giant creature. Use bows, crossbows, traps or suddenly spring from the bush and use a large axe (not a sword, you need the force and cutting power of an axe, there's a reason we don't cut trees with swords), then try to hide in the brush again. The giant will take a while, but he's much more likely to bleed to death.




        • Viral warfare


        Poisons might not be that readily available, a pit with multi-week old poop from different species as well as some dead critters will (2). Dip your weapons into that before you wound the giant. You could also put that stuff into food you put out in the forest, for example some dead humans to lure them in (criminals if it makes your conscience better, or if you don't have a conscience, use live ones).



        (1) source as offered by Douwe: https://www.theverge.com/tldr/2016/6/15/11936802/ant-dropped-from-empire-state-building-science-experiment-mystery-solved



        (2) https://en.m.wikipedia.org/wiki/History_of_biological_warfare






        share|improve this answer











        $endgroup$









        • 7




          $begingroup$
          Drop an ant from an airplane and it lands safely on the ground Accurate apparently, sorry for doubting you :)
          $endgroup$
          – Douwe
          Jan 8 at 12:17








        • 1




          $begingroup$
          The last one was an actual medieval tactic used to contaminate blades and arrows. To avoid dripping a (reusable) sword blade in the nasty stuff, they could also suspend it just above it for the night.
          $endgroup$
          – Eth
          Jan 8 at 12:58










        • $begingroup$
          @Douwe no problem. Many thanks for looking up sources and posting them. I'll add it to the answer.
          $endgroup$
          – Demigan
          Jan 8 at 14:20






        • 1




          $begingroup$
          "A rat is killed, a man is broken, a horse splashes." - JBS Haldane
          $endgroup$
          – Arcanist Lupus
          Jan 8 at 14:41






        • 1




          $begingroup$
          @Yakk No matter the pressure, the trap needs to be large enough for the foot to actually fall into. And then, the entire weight of the giant rests on the trap when it's supposed to give in. Thus, the supporting structures that are supposed to give in at about 343*70kg = 2.4t can easily discern this weight even from a few hundreds of kg. There is absolutely no need to design the trap to support 343 humans.
          $endgroup$
          – cmaster
          Jan 10 at 21:21














        21












        21








        21





        $begingroup$


        • Your first weapon is a shovel.


        Drop an ant from an airplane and it lands safely on the ground (1), drop an elephant from 2 meters height and it breaks all its bones. Drop a giant... You simply dig traps, cover them up preferably in a way that people at people height can see warnings and a giant can't and wait. Even relatively shallow holes will break the arms and torso of the giant, if not his legs as well. This might not be a totally valid answer if you consider that the giant is surviving the square-cube law. As an alternative he could use a lot of thick ropes strung up around trees to trip the giant.




        • Make it bleed.


        To pump all that blood around the giant is going to need a lot more blood pressure, and a cut on the leg for example is going to be much harder to stop bleeding for such a giant creature. Use bows, crossbows, traps or suddenly spring from the bush and use a large axe (not a sword, you need the force and cutting power of an axe, there's a reason we don't cut trees with swords), then try to hide in the brush again. The giant will take a while, but he's much more likely to bleed to death.




        • Viral warfare


        Poisons might not be that readily available, a pit with multi-week old poop from different species as well as some dead critters will (2). Dip your weapons into that before you wound the giant. You could also put that stuff into food you put out in the forest, for example some dead humans to lure them in (criminals if it makes your conscience better, or if you don't have a conscience, use live ones).



        (1) source as offered by Douwe: https://www.theverge.com/tldr/2016/6/15/11936802/ant-dropped-from-empire-state-building-science-experiment-mystery-solved



        (2) https://en.m.wikipedia.org/wiki/History_of_biological_warfare






        share|improve this answer











        $endgroup$




        • Your first weapon is a shovel.


        Drop an ant from an airplane and it lands safely on the ground (1), drop an elephant from 2 meters height and it breaks all its bones. Drop a giant... You simply dig traps, cover them up preferably in a way that people at people height can see warnings and a giant can't and wait. Even relatively shallow holes will break the arms and torso of the giant, if not his legs as well. This might not be a totally valid answer if you consider that the giant is surviving the square-cube law. As an alternative he could use a lot of thick ropes strung up around trees to trip the giant.




        • Make it bleed.


        To pump all that blood around the giant is going to need a lot more blood pressure, and a cut on the leg for example is going to be much harder to stop bleeding for such a giant creature. Use bows, crossbows, traps or suddenly spring from the bush and use a large axe (not a sword, you need the force and cutting power of an axe, there's a reason we don't cut trees with swords), then try to hide in the brush again. The giant will take a while, but he's much more likely to bleed to death.




        • Viral warfare


        Poisons might not be that readily available, a pit with multi-week old poop from different species as well as some dead critters will (2). Dip your weapons into that before you wound the giant. You could also put that stuff into food you put out in the forest, for example some dead humans to lure them in (criminals if it makes your conscience better, or if you don't have a conscience, use live ones).



        (1) source as offered by Douwe: https://www.theverge.com/tldr/2016/6/15/11936802/ant-dropped-from-empire-state-building-science-experiment-mystery-solved



        (2) https://en.m.wikipedia.org/wiki/History_of_biological_warfare







        share|improve this answer














        share|improve this answer



        share|improve this answer








        edited Jan 8 at 21:20

























        answered Jan 8 at 8:00









        DemiganDemigan

        8,0001741




        8,0001741








        • 7




          $begingroup$
          Drop an ant from an airplane and it lands safely on the ground Accurate apparently, sorry for doubting you :)
          $endgroup$
          – Douwe
          Jan 8 at 12:17








        • 1




          $begingroup$
          The last one was an actual medieval tactic used to contaminate blades and arrows. To avoid dripping a (reusable) sword blade in the nasty stuff, they could also suspend it just above it for the night.
          $endgroup$
          – Eth
          Jan 8 at 12:58










        • $begingroup$
          @Douwe no problem. Many thanks for looking up sources and posting them. I'll add it to the answer.
          $endgroup$
          – Demigan
          Jan 8 at 14:20






        • 1




          $begingroup$
          "A rat is killed, a man is broken, a horse splashes." - JBS Haldane
          $endgroup$
          – Arcanist Lupus
          Jan 8 at 14:41






        • 1




          $begingroup$
          @Yakk No matter the pressure, the trap needs to be large enough for the foot to actually fall into. And then, the entire weight of the giant rests on the trap when it's supposed to give in. Thus, the supporting structures that are supposed to give in at about 343*70kg = 2.4t can easily discern this weight even from a few hundreds of kg. There is absolutely no need to design the trap to support 343 humans.
          $endgroup$
          – cmaster
          Jan 10 at 21:21














        • 7




          $begingroup$
          Drop an ant from an airplane and it lands safely on the ground Accurate apparently, sorry for doubting you :)
          $endgroup$
          – Douwe
          Jan 8 at 12:17








        • 1




          $begingroup$
          The last one was an actual medieval tactic used to contaminate blades and arrows. To avoid dripping a (reusable) sword blade in the nasty stuff, they could also suspend it just above it for the night.
          $endgroup$
          – Eth
          Jan 8 at 12:58










        • $begingroup$
          @Douwe no problem. Many thanks for looking up sources and posting them. I'll add it to the answer.
          $endgroup$
          – Demigan
          Jan 8 at 14:20






        • 1




          $begingroup$
          "A rat is killed, a man is broken, a horse splashes." - JBS Haldane
          $endgroup$
          – Arcanist Lupus
          Jan 8 at 14:41






        • 1




          $begingroup$
          @Yakk No matter the pressure, the trap needs to be large enough for the foot to actually fall into. And then, the entire weight of the giant rests on the trap when it's supposed to give in. Thus, the supporting structures that are supposed to give in at about 343*70kg = 2.4t can easily discern this weight even from a few hundreds of kg. There is absolutely no need to design the trap to support 343 humans.
          $endgroup$
          – cmaster
          Jan 10 at 21:21








        7




        7




        $begingroup$
        Drop an ant from an airplane and it lands safely on the ground Accurate apparently, sorry for doubting you :)
        $endgroup$
        – Douwe
        Jan 8 at 12:17






        $begingroup$
        Drop an ant from an airplane and it lands safely on the ground Accurate apparently, sorry for doubting you :)
        $endgroup$
        – Douwe
        Jan 8 at 12:17






        1




        1




        $begingroup$
        The last one was an actual medieval tactic used to contaminate blades and arrows. To avoid dripping a (reusable) sword blade in the nasty stuff, they could also suspend it just above it for the night.
        $endgroup$
        – Eth
        Jan 8 at 12:58




        $begingroup$
        The last one was an actual medieval tactic used to contaminate blades and arrows. To avoid dripping a (reusable) sword blade in the nasty stuff, they could also suspend it just above it for the night.
        $endgroup$
        – Eth
        Jan 8 at 12:58












        $begingroup$
        @Douwe no problem. Many thanks for looking up sources and posting them. I'll add it to the answer.
        $endgroup$
        – Demigan
        Jan 8 at 14:20




        $begingroup$
        @Douwe no problem. Many thanks for looking up sources and posting them. I'll add it to the answer.
        $endgroup$
        – Demigan
        Jan 8 at 14:20




        1




        1




        $begingroup$
        "A rat is killed, a man is broken, a horse splashes." - JBS Haldane
        $endgroup$
        – Arcanist Lupus
        Jan 8 at 14:41




        $begingroup$
        "A rat is killed, a man is broken, a horse splashes." - JBS Haldane
        $endgroup$
        – Arcanist Lupus
        Jan 8 at 14:41




        1




        1




        $begingroup$
        @Yakk No matter the pressure, the trap needs to be large enough for the foot to actually fall into. And then, the entire weight of the giant rests on the trap when it's supposed to give in. Thus, the supporting structures that are supposed to give in at about 343*70kg = 2.4t can easily discern this weight even from a few hundreds of kg. There is absolutely no need to design the trap to support 343 humans.
        $endgroup$
        – cmaster
        Jan 10 at 21:21




        $begingroup$
        @Yakk No matter the pressure, the trap needs to be large enough for the foot to actually fall into. And then, the entire weight of the giant rests on the trap when it's supposed to give in. Thus, the supporting structures that are supposed to give in at about 343*70kg = 2.4t can easily discern this weight even from a few hundreds of kg. There is absolutely no need to design the trap to support 343 humans.
        $endgroup$
        – cmaster
        Jan 10 at 21:21











        9












        $begingroup$

        Since the giants are slow and strong, I would recommend just running around naked. That will make your hunter extremely light and nimble and so that they can recover from their I-Frame dodge rolls quickly. Armor is of little use if the creature your fighting is both 7 times larger and stronger. Nothing is going to really protect your from one of their attacks, so armor should really just be clothes based on the environment you are hunting in.



        As for the weapon, I would recommend poison. There are many lethal poisons out there that even in small doses would kill a giant. So while a drop might kill several humans, a couple more drops will kill a giant. And since they are slow, you just stab them with a needle several times (the needles size will have to vary depending on the giants skin thickness and toughness) and run away and wait for the poison to do its work.






        share|improve this answer









        $endgroup$









        • 1




          $begingroup$
          Poisoned rapier, perhaps?
          $endgroup$
          – Arkenstein XII
          Jan 8 at 2:14






        • 1




          $begingroup$
          Well, you do have to consider how thick their skin is, small poisonous spiders can bite humans, but their fangs might not even penetrate our skin. Adult spiders, however are a different issue.
          $endgroup$
          – Mr.J
          Jan 8 at 3:21






        • 4




          $begingroup$
          a slingshot springs to mind.
          $endgroup$
          – Giu Piete
          Jan 8 at 5:42










        • $begingroup$
          /I would recommend just running around naked/. You answer every single question that way! Not saying you're wrong, of course.
          $endgroup$
          – Willk
          Jan 9 at 1:11












        • $begingroup$
          @Willk I've been playing dark souls lately... need to remove all my equipment for the quick roll. Gloves and boots are all the protection I need.
          $endgroup$
          – Shadowzee
          Jan 9 at 1:13
















        9












        $begingroup$

        Since the giants are slow and strong, I would recommend just running around naked. That will make your hunter extremely light and nimble and so that they can recover from their I-Frame dodge rolls quickly. Armor is of little use if the creature your fighting is both 7 times larger and stronger. Nothing is going to really protect your from one of their attacks, so armor should really just be clothes based on the environment you are hunting in.



        As for the weapon, I would recommend poison. There are many lethal poisons out there that even in small doses would kill a giant. So while a drop might kill several humans, a couple more drops will kill a giant. And since they are slow, you just stab them with a needle several times (the needles size will have to vary depending on the giants skin thickness and toughness) and run away and wait for the poison to do its work.






        share|improve this answer









        $endgroup$









        • 1




          $begingroup$
          Poisoned rapier, perhaps?
          $endgroup$
          – Arkenstein XII
          Jan 8 at 2:14






        • 1




          $begingroup$
          Well, you do have to consider how thick their skin is, small poisonous spiders can bite humans, but their fangs might not even penetrate our skin. Adult spiders, however are a different issue.
          $endgroup$
          – Mr.J
          Jan 8 at 3:21






        • 4




          $begingroup$
          a slingshot springs to mind.
          $endgroup$
          – Giu Piete
          Jan 8 at 5:42










        • $begingroup$
          /I would recommend just running around naked/. You answer every single question that way! Not saying you're wrong, of course.
          $endgroup$
          – Willk
          Jan 9 at 1:11












        • $begingroup$
          @Willk I've been playing dark souls lately... need to remove all my equipment for the quick roll. Gloves and boots are all the protection I need.
          $endgroup$
          – Shadowzee
          Jan 9 at 1:13














        9












        9








        9





        $begingroup$

        Since the giants are slow and strong, I would recommend just running around naked. That will make your hunter extremely light and nimble and so that they can recover from their I-Frame dodge rolls quickly. Armor is of little use if the creature your fighting is both 7 times larger and stronger. Nothing is going to really protect your from one of their attacks, so armor should really just be clothes based on the environment you are hunting in.



        As for the weapon, I would recommend poison. There are many lethal poisons out there that even in small doses would kill a giant. So while a drop might kill several humans, a couple more drops will kill a giant. And since they are slow, you just stab them with a needle several times (the needles size will have to vary depending on the giants skin thickness and toughness) and run away and wait for the poison to do its work.






        share|improve this answer









        $endgroup$



        Since the giants are slow and strong, I would recommend just running around naked. That will make your hunter extremely light and nimble and so that they can recover from their I-Frame dodge rolls quickly. Armor is of little use if the creature your fighting is both 7 times larger and stronger. Nothing is going to really protect your from one of their attacks, so armor should really just be clothes based on the environment you are hunting in.



        As for the weapon, I would recommend poison. There are many lethal poisons out there that even in small doses would kill a giant. So while a drop might kill several humans, a couple more drops will kill a giant. And since they are slow, you just stab them with a needle several times (the needles size will have to vary depending on the giants skin thickness and toughness) and run away and wait for the poison to do its work.







        share|improve this answer












        share|improve this answer



        share|improve this answer










        answered Jan 8 at 2:09









        ShadowzeeShadowzee

        7,5501236




        7,5501236








        • 1




          $begingroup$
          Poisoned rapier, perhaps?
          $endgroup$
          – Arkenstein XII
          Jan 8 at 2:14






        • 1




          $begingroup$
          Well, you do have to consider how thick their skin is, small poisonous spiders can bite humans, but their fangs might not even penetrate our skin. Adult spiders, however are a different issue.
          $endgroup$
          – Mr.J
          Jan 8 at 3:21






        • 4




          $begingroup$
          a slingshot springs to mind.
          $endgroup$
          – Giu Piete
          Jan 8 at 5:42










        • $begingroup$
          /I would recommend just running around naked/. You answer every single question that way! Not saying you're wrong, of course.
          $endgroup$
          – Willk
          Jan 9 at 1:11












        • $begingroup$
          @Willk I've been playing dark souls lately... need to remove all my equipment for the quick roll. Gloves and boots are all the protection I need.
          $endgroup$
          – Shadowzee
          Jan 9 at 1:13














        • 1




          $begingroup$
          Poisoned rapier, perhaps?
          $endgroup$
          – Arkenstein XII
          Jan 8 at 2:14






        • 1




          $begingroup$
          Well, you do have to consider how thick their skin is, small poisonous spiders can bite humans, but their fangs might not even penetrate our skin. Adult spiders, however are a different issue.
          $endgroup$
          – Mr.J
          Jan 8 at 3:21






        • 4




          $begingroup$
          a slingshot springs to mind.
          $endgroup$
          – Giu Piete
          Jan 8 at 5:42










        • $begingroup$
          /I would recommend just running around naked/. You answer every single question that way! Not saying you're wrong, of course.
          $endgroup$
          – Willk
          Jan 9 at 1:11












        • $begingroup$
          @Willk I've been playing dark souls lately... need to remove all my equipment for the quick roll. Gloves and boots are all the protection I need.
          $endgroup$
          – Shadowzee
          Jan 9 at 1:13








        1




        1




        $begingroup$
        Poisoned rapier, perhaps?
        $endgroup$
        – Arkenstein XII
        Jan 8 at 2:14




        $begingroup$
        Poisoned rapier, perhaps?
        $endgroup$
        – Arkenstein XII
        Jan 8 at 2:14




        1




        1




        $begingroup$
        Well, you do have to consider how thick their skin is, small poisonous spiders can bite humans, but their fangs might not even penetrate our skin. Adult spiders, however are a different issue.
        $endgroup$
        – Mr.J
        Jan 8 at 3:21




        $begingroup$
        Well, you do have to consider how thick their skin is, small poisonous spiders can bite humans, but their fangs might not even penetrate our skin. Adult spiders, however are a different issue.
        $endgroup$
        – Mr.J
        Jan 8 at 3:21




        4




        4




        $begingroup$
        a slingshot springs to mind.
        $endgroup$
        – Giu Piete
        Jan 8 at 5:42




        $begingroup$
        a slingshot springs to mind.
        $endgroup$
        – Giu Piete
        Jan 8 at 5:42












        $begingroup$
        /I would recommend just running around naked/. You answer every single question that way! Not saying you're wrong, of course.
        $endgroup$
        – Willk
        Jan 9 at 1:11






        $begingroup$
        /I would recommend just running around naked/. You answer every single question that way! Not saying you're wrong, of course.
        $endgroup$
        – Willk
        Jan 9 at 1:11














        $begingroup$
        @Willk I've been playing dark souls lately... need to remove all my equipment for the quick roll. Gloves and boots are all the protection I need.
        $endgroup$
        – Shadowzee
        Jan 9 at 1:13




        $begingroup$
        @Willk I've been playing dark souls lately... need to remove all my equipment for the quick roll. Gloves and boots are all the protection I need.
        $endgroup$
        – Shadowzee
        Jan 9 at 1:13











        7












        $begingroup$

        The smart path would be to use a crossbow or longbow and to shoot the giants with poisoned arrows while on horseback (He does not have to shoot on the move. He could just kite the giants by shooting and then moving away, then shooting again).



        Keep in mind that the giants would probably be able to throw rocks at him, so he would have to maintain some distance. Also, the giants are quite large, so it would take a lot of poison to kill them. The smart thing to do would be for him to have several people accompany him.



        He would want to avoid melee at any cost, as even a glancing blow from a giant could smash his head in, no matter what armor he wears. Digging pit traps with feces coated stakes on the bottom and luring giants into them could be viable. Carrying some caltrops and scattering them behind him might be useful, but the giant might just walk around those. They would be quite effective at night.



        Also, he might want to wear at least a gambeson. Even if it would not protect against melee strikes or thrown rocks, smarter giants might throw clay pots full of small stones at him like hand grenades, and he would want some protection from that.






        share|improve this answer









        $endgroup$













        • $begingroup$
          If you don't have to kill the giant quickly, consider disease rather than poison - biological agents multiply, so a small dose will grow inside the giant until reaching lethal levels...
          $endgroup$
          – G0BLiN
          Jan 8 at 12:38






        • 1




          $begingroup$
          I totally forgot about the magnetic fields.
          $endgroup$
          – Richard Smith
          Jan 8 at 15:36
















        7












        $begingroup$

        The smart path would be to use a crossbow or longbow and to shoot the giants with poisoned arrows while on horseback (He does not have to shoot on the move. He could just kite the giants by shooting and then moving away, then shooting again).



        Keep in mind that the giants would probably be able to throw rocks at him, so he would have to maintain some distance. Also, the giants are quite large, so it would take a lot of poison to kill them. The smart thing to do would be for him to have several people accompany him.



        He would want to avoid melee at any cost, as even a glancing blow from a giant could smash his head in, no matter what armor he wears. Digging pit traps with feces coated stakes on the bottom and luring giants into them could be viable. Carrying some caltrops and scattering them behind him might be useful, but the giant might just walk around those. They would be quite effective at night.



        Also, he might want to wear at least a gambeson. Even if it would not protect against melee strikes or thrown rocks, smarter giants might throw clay pots full of small stones at him like hand grenades, and he would want some protection from that.






        share|improve this answer









        $endgroup$













        • $begingroup$
          If you don't have to kill the giant quickly, consider disease rather than poison - biological agents multiply, so a small dose will grow inside the giant until reaching lethal levels...
          $endgroup$
          – G0BLiN
          Jan 8 at 12:38






        • 1




          $begingroup$
          I totally forgot about the magnetic fields.
          $endgroup$
          – Richard Smith
          Jan 8 at 15:36














        7












        7








        7





        $begingroup$

        The smart path would be to use a crossbow or longbow and to shoot the giants with poisoned arrows while on horseback (He does not have to shoot on the move. He could just kite the giants by shooting and then moving away, then shooting again).



        Keep in mind that the giants would probably be able to throw rocks at him, so he would have to maintain some distance. Also, the giants are quite large, so it would take a lot of poison to kill them. The smart thing to do would be for him to have several people accompany him.



        He would want to avoid melee at any cost, as even a glancing blow from a giant could smash his head in, no matter what armor he wears. Digging pit traps with feces coated stakes on the bottom and luring giants into them could be viable. Carrying some caltrops and scattering them behind him might be useful, but the giant might just walk around those. They would be quite effective at night.



        Also, he might want to wear at least a gambeson. Even if it would not protect against melee strikes or thrown rocks, smarter giants might throw clay pots full of small stones at him like hand grenades, and he would want some protection from that.






        share|improve this answer









        $endgroup$



        The smart path would be to use a crossbow or longbow and to shoot the giants with poisoned arrows while on horseback (He does not have to shoot on the move. He could just kite the giants by shooting and then moving away, then shooting again).



        Keep in mind that the giants would probably be able to throw rocks at him, so he would have to maintain some distance. Also, the giants are quite large, so it would take a lot of poison to kill them. The smart thing to do would be for him to have several people accompany him.



        He would want to avoid melee at any cost, as even a glancing blow from a giant could smash his head in, no matter what armor he wears. Digging pit traps with feces coated stakes on the bottom and luring giants into them could be viable. Carrying some caltrops and scattering them behind him might be useful, but the giant might just walk around those. They would be quite effective at night.



        Also, he might want to wear at least a gambeson. Even if it would not protect against melee strikes or thrown rocks, smarter giants might throw clay pots full of small stones at him like hand grenades, and he would want some protection from that.







        share|improve this answer












        share|improve this answer



        share|improve this answer










        answered Jan 8 at 2:40









        Richard SmithRichard Smith

        52029




        52029












        • $begingroup$
          If you don't have to kill the giant quickly, consider disease rather than poison - biological agents multiply, so a small dose will grow inside the giant until reaching lethal levels...
          $endgroup$
          – G0BLiN
          Jan 8 at 12:38






        • 1




          $begingroup$
          I totally forgot about the magnetic fields.
          $endgroup$
          – Richard Smith
          Jan 8 at 15:36


















        • $begingroup$
          If you don't have to kill the giant quickly, consider disease rather than poison - biological agents multiply, so a small dose will grow inside the giant until reaching lethal levels...
          $endgroup$
          – G0BLiN
          Jan 8 at 12:38






        • 1




          $begingroup$
          I totally forgot about the magnetic fields.
          $endgroup$
          – Richard Smith
          Jan 8 at 15:36
















        $begingroup$
        If you don't have to kill the giant quickly, consider disease rather than poison - biological agents multiply, so a small dose will grow inside the giant until reaching lethal levels...
        $endgroup$
        – G0BLiN
        Jan 8 at 12:38




        $begingroup$
        If you don't have to kill the giant quickly, consider disease rather than poison - biological agents multiply, so a small dose will grow inside the giant until reaching lethal levels...
        $endgroup$
        – G0BLiN
        Jan 8 at 12:38




        1




        1




        $begingroup$
        I totally forgot about the magnetic fields.
        $endgroup$
        – Richard Smith
        Jan 8 at 15:36




        $begingroup$
        I totally forgot about the magnetic fields.
        $endgroup$
        – Richard Smith
        Jan 8 at 15:36











        3












        $begingroup$

        Fighting one or two giants are easy, but fighting a group? that's one thing to consider.



        You can't become a "Giant slayer" just because you killed one, you have to have killed a plenty.



        That being said, I do think giant killer is a wrong title as your giants are WAAAAAY to strong and WAAAAY to tall for someone to kill with a blade, even with poison. At that height, the most weakest point of the giant is the feet. big, range weapons will penetrate those but they are a burden to carry by one man.



        A more plausible approach might be, The Giant killing army.



        Your army can, scatter long, steal spike, drench them with poison, then cover them with forestry.



        Your main weapon for killing them will be ballistas... Lots of them, as they are the weapon with the highest percentage of killing a giant without any casualties.



        There should be one improvement however, you ballista must have a reloading mechanism similar to a Van Helsing's repeater crossbow because you need to reload as quickly as possible, but pack a punch too.



        one of your problems however, is that you got 100 persons, give or take that some of them are women, or grandparents and kids, your army lacks manpower, 50 might hold for some time.



        I also suggest to be on the defensive rather than taking the fight too them, setting up traps is the best way to kill the giants, you need all the manpower to protect your people and ensure continuity of the human race.



        another problem is giant carcass, yeah you killed them, but if they die close to your encampment then then stench will kill you as well, so you better plan your traps, how you set your encampment, and how you can engage the giants with your ballistas on the encampment, and during the move.



        you will have a pretty busy life if you ask me...






        share|improve this answer









        $endgroup$


















          3












          $begingroup$

          Fighting one or two giants are easy, but fighting a group? that's one thing to consider.



          You can't become a "Giant slayer" just because you killed one, you have to have killed a plenty.



          That being said, I do think giant killer is a wrong title as your giants are WAAAAAY to strong and WAAAAY to tall for someone to kill with a blade, even with poison. At that height, the most weakest point of the giant is the feet. big, range weapons will penetrate those but they are a burden to carry by one man.



          A more plausible approach might be, The Giant killing army.



          Your army can, scatter long, steal spike, drench them with poison, then cover them with forestry.



          Your main weapon for killing them will be ballistas... Lots of them, as they are the weapon with the highest percentage of killing a giant without any casualties.



          There should be one improvement however, you ballista must have a reloading mechanism similar to a Van Helsing's repeater crossbow because you need to reload as quickly as possible, but pack a punch too.



          one of your problems however, is that you got 100 persons, give or take that some of them are women, or grandparents and kids, your army lacks manpower, 50 might hold for some time.



          I also suggest to be on the defensive rather than taking the fight too them, setting up traps is the best way to kill the giants, you need all the manpower to protect your people and ensure continuity of the human race.



          another problem is giant carcass, yeah you killed them, but if they die close to your encampment then then stench will kill you as well, so you better plan your traps, how you set your encampment, and how you can engage the giants with your ballistas on the encampment, and during the move.



          you will have a pretty busy life if you ask me...






          share|improve this answer









          $endgroup$
















            3












            3








            3





            $begingroup$

            Fighting one or two giants are easy, but fighting a group? that's one thing to consider.



            You can't become a "Giant slayer" just because you killed one, you have to have killed a plenty.



            That being said, I do think giant killer is a wrong title as your giants are WAAAAAY to strong and WAAAAY to tall for someone to kill with a blade, even with poison. At that height, the most weakest point of the giant is the feet. big, range weapons will penetrate those but they are a burden to carry by one man.



            A more plausible approach might be, The Giant killing army.



            Your army can, scatter long, steal spike, drench them with poison, then cover them with forestry.



            Your main weapon for killing them will be ballistas... Lots of them, as they are the weapon with the highest percentage of killing a giant without any casualties.



            There should be one improvement however, you ballista must have a reloading mechanism similar to a Van Helsing's repeater crossbow because you need to reload as quickly as possible, but pack a punch too.



            one of your problems however, is that you got 100 persons, give or take that some of them are women, or grandparents and kids, your army lacks manpower, 50 might hold for some time.



            I also suggest to be on the defensive rather than taking the fight too them, setting up traps is the best way to kill the giants, you need all the manpower to protect your people and ensure continuity of the human race.



            another problem is giant carcass, yeah you killed them, but if they die close to your encampment then then stench will kill you as well, so you better plan your traps, how you set your encampment, and how you can engage the giants with your ballistas on the encampment, and during the move.



            you will have a pretty busy life if you ask me...






            share|improve this answer









            $endgroup$



            Fighting one or two giants are easy, but fighting a group? that's one thing to consider.



            You can't become a "Giant slayer" just because you killed one, you have to have killed a plenty.



            That being said, I do think giant killer is a wrong title as your giants are WAAAAAY to strong and WAAAAY to tall for someone to kill with a blade, even with poison. At that height, the most weakest point of the giant is the feet. big, range weapons will penetrate those but they are a burden to carry by one man.



            A more plausible approach might be, The Giant killing army.



            Your army can, scatter long, steal spike, drench them with poison, then cover them with forestry.



            Your main weapon for killing them will be ballistas... Lots of them, as they are the weapon with the highest percentage of killing a giant without any casualties.



            There should be one improvement however, you ballista must have a reloading mechanism similar to a Van Helsing's repeater crossbow because you need to reload as quickly as possible, but pack a punch too.



            one of your problems however, is that you got 100 persons, give or take that some of them are women, or grandparents and kids, your army lacks manpower, 50 might hold for some time.



            I also suggest to be on the defensive rather than taking the fight too them, setting up traps is the best way to kill the giants, you need all the manpower to protect your people and ensure continuity of the human race.



            another problem is giant carcass, yeah you killed them, but if they die close to your encampment then then stench will kill you as well, so you better plan your traps, how you set your encampment, and how you can engage the giants with your ballistas on the encampment, and during the move.



            you will have a pretty busy life if you ask me...







            share|improve this answer












            share|improve this answer



            share|improve this answer










            answered Jan 8 at 5:28









            Mr.JMr.J

            1,651629




            1,651629























                3












                $begingroup$

                "He is able to bend and generate magnetic fields" - Does it mean he can manipulate metal and utilize it for combat? Well... in that case "Avatar: The Last Airbender" comes to mind with metal benders (rather less accurate, meaning it's more like throwing and shooting metallic objects), or X-Man with Magneto (full, precise control), or even Irelia (special weapon) in League of Legends.



                In both cases he'd carry large blades around, either swords or just the blade parts. When he fights he swings them around on range, pierces his opponents, performs attacks from their backs (when returning the blades). Against giants he would go for the weak spots, which he can reach unlike normal humans.



                For armor he'd either wear leather (in case the metal interferes with his abilities if it covers his body) or an armor with metallic parts, which he can optionally use as weapons or as defence. Alternatively his armor could consist of the weapons he is carrying around, meaning the blades are part of the armor (the shoulder pads or on the backside).






                share|improve this answer











                $endgroup$


















                  3












                  $begingroup$

                  "He is able to bend and generate magnetic fields" - Does it mean he can manipulate metal and utilize it for combat? Well... in that case "Avatar: The Last Airbender" comes to mind with metal benders (rather less accurate, meaning it's more like throwing and shooting metallic objects), or X-Man with Magneto (full, precise control), or even Irelia (special weapon) in League of Legends.



                  In both cases he'd carry large blades around, either swords or just the blade parts. When he fights he swings them around on range, pierces his opponents, performs attacks from their backs (when returning the blades). Against giants he would go for the weak spots, which he can reach unlike normal humans.



                  For armor he'd either wear leather (in case the metal interferes with his abilities if it covers his body) or an armor with metallic parts, which he can optionally use as weapons or as defence. Alternatively his armor could consist of the weapons he is carrying around, meaning the blades are part of the armor (the shoulder pads or on the backside).






                  share|improve this answer











                  $endgroup$
















                    3












                    3








                    3





                    $begingroup$

                    "He is able to bend and generate magnetic fields" - Does it mean he can manipulate metal and utilize it for combat? Well... in that case "Avatar: The Last Airbender" comes to mind with metal benders (rather less accurate, meaning it's more like throwing and shooting metallic objects), or X-Man with Magneto (full, precise control), or even Irelia (special weapon) in League of Legends.



                    In both cases he'd carry large blades around, either swords or just the blade parts. When he fights he swings them around on range, pierces his opponents, performs attacks from their backs (when returning the blades). Against giants he would go for the weak spots, which he can reach unlike normal humans.



                    For armor he'd either wear leather (in case the metal interferes with his abilities if it covers his body) or an armor with metallic parts, which he can optionally use as weapons or as defence. Alternatively his armor could consist of the weapons he is carrying around, meaning the blades are part of the armor (the shoulder pads or on the backside).






                    share|improve this answer











                    $endgroup$



                    "He is able to bend and generate magnetic fields" - Does it mean he can manipulate metal and utilize it for combat? Well... in that case "Avatar: The Last Airbender" comes to mind with metal benders (rather less accurate, meaning it's more like throwing and shooting metallic objects), or X-Man with Magneto (full, precise control), or even Irelia (special weapon) in League of Legends.



                    In both cases he'd carry large blades around, either swords or just the blade parts. When he fights he swings them around on range, pierces his opponents, performs attacks from their backs (when returning the blades). Against giants he would go for the weak spots, which he can reach unlike normal humans.



                    For armor he'd either wear leather (in case the metal interferes with his abilities if it covers his body) or an armor with metallic parts, which he can optionally use as weapons or as defence. Alternatively his armor could consist of the weapons he is carrying around, meaning the blades are part of the armor (the shoulder pads or on the backside).







                    share|improve this answer














                    share|improve this answer



                    share|improve this answer








                    edited Jan 8 at 7:18

























                    answered Jan 8 at 7:08









                    BattleBattle

                    78111




                    78111























                        2












                        $begingroup$

                        Rope. Think of the AT-AT's in Star Wars, just run around it and try to trip it. Other similar things would probably work also like nets. Once it is slowed down/downed, just stick it with the pointy end.



                        If they are slow enough, maybe bring a large crossbow/ballista mounted on a carriage or something.



                        Metallic spike traps, wall or ground mounted if they can manipulated by magnetic force, depending on strength of manipulation.
                        enter image description here






                        share|improve this answer











                        $endgroup$













                        • $begingroup$
                          Metallic bolas might work better for a person capable of manipulating magnetic fields... :)
                          $endgroup$
                          – G0BLiN
                          Jan 8 at 12:41
















                        2












                        $begingroup$

                        Rope. Think of the AT-AT's in Star Wars, just run around it and try to trip it. Other similar things would probably work also like nets. Once it is slowed down/downed, just stick it with the pointy end.



                        If they are slow enough, maybe bring a large crossbow/ballista mounted on a carriage or something.



                        Metallic spike traps, wall or ground mounted if they can manipulated by magnetic force, depending on strength of manipulation.
                        enter image description here






                        share|improve this answer











                        $endgroup$













                        • $begingroup$
                          Metallic bolas might work better for a person capable of manipulating magnetic fields... :)
                          $endgroup$
                          – G0BLiN
                          Jan 8 at 12:41














                        2












                        2








                        2





                        $begingroup$

                        Rope. Think of the AT-AT's in Star Wars, just run around it and try to trip it. Other similar things would probably work also like nets. Once it is slowed down/downed, just stick it with the pointy end.



                        If they are slow enough, maybe bring a large crossbow/ballista mounted on a carriage or something.



                        Metallic spike traps, wall or ground mounted if they can manipulated by magnetic force, depending on strength of manipulation.
                        enter image description here






                        share|improve this answer











                        $endgroup$



                        Rope. Think of the AT-AT's in Star Wars, just run around it and try to trip it. Other similar things would probably work also like nets. Once it is slowed down/downed, just stick it with the pointy end.



                        If they are slow enough, maybe bring a large crossbow/ballista mounted on a carriage or something.



                        Metallic spike traps, wall or ground mounted if they can manipulated by magnetic force, depending on strength of manipulation.
                        enter image description here







                        share|improve this answer














                        share|improve this answer



                        share|improve this answer








                        edited Jan 8 at 12:47

























                        answered Jan 8 at 9:28









                        Viktor MellgrenViktor Mellgren

                        381411




                        381411












                        • $begingroup$
                          Metallic bolas might work better for a person capable of manipulating magnetic fields... :)
                          $endgroup$
                          – G0BLiN
                          Jan 8 at 12:41


















                        • $begingroup$
                          Metallic bolas might work better for a person capable of manipulating magnetic fields... :)
                          $endgroup$
                          – G0BLiN
                          Jan 8 at 12:41
















                        $begingroup$
                        Metallic bolas might work better for a person capable of manipulating magnetic fields... :)
                        $endgroup$
                        – G0BLiN
                        Jan 8 at 12:41




                        $begingroup$
                        Metallic bolas might work better for a person capable of manipulating magnetic fields... :)
                        $endgroup$
                        – G0BLiN
                        Jan 8 at 12:41











                        1












                        $begingroup$

                        Science



                        If there are only 100 giants around at any time, then they are at the brink of extinction. The amount of inbreeding over generations is astounding. The giants are probably stupid to the extreme due to that. Also, with only 100 giants at any time they will hardly be able to keep a culture or be educated.



                        Sinxe they hunt by smell, all the slayer needs to do is dressing scarecrows with clothes recently worn by actual people. Then fill the scarecrows with fresh but poisoned meat. Just wait for a few days before checking for each scarecrow.






                        share|improve this answer









                        $endgroup$













                        • $begingroup$
                          indeed they are stupid. They will be later controlled with the use of an artifact and this dude will fight them because the one controlling them want to get over a territory which includes his home place.
                          $endgroup$
                          – Edward Constantin
                          Jan 8 at 11:12
















                        1












                        $begingroup$

                        Science



                        If there are only 100 giants around at any time, then they are at the brink of extinction. The amount of inbreeding over generations is astounding. The giants are probably stupid to the extreme due to that. Also, with only 100 giants at any time they will hardly be able to keep a culture or be educated.



                        Sinxe they hunt by smell, all the slayer needs to do is dressing scarecrows with clothes recently worn by actual people. Then fill the scarecrows with fresh but poisoned meat. Just wait for a few days before checking for each scarecrow.






                        share|improve this answer









                        $endgroup$













                        • $begingroup$
                          indeed they are stupid. They will be later controlled with the use of an artifact and this dude will fight them because the one controlling them want to get over a territory which includes his home place.
                          $endgroup$
                          – Edward Constantin
                          Jan 8 at 11:12














                        1












                        1








                        1





                        $begingroup$

                        Science



                        If there are only 100 giants around at any time, then they are at the brink of extinction. The amount of inbreeding over generations is astounding. The giants are probably stupid to the extreme due to that. Also, with only 100 giants at any time they will hardly be able to keep a culture or be educated.



                        Sinxe they hunt by smell, all the slayer needs to do is dressing scarecrows with clothes recently worn by actual people. Then fill the scarecrows with fresh but poisoned meat. Just wait for a few days before checking for each scarecrow.






                        share|improve this answer









                        $endgroup$



                        Science



                        If there are only 100 giants around at any time, then they are at the brink of extinction. The amount of inbreeding over generations is astounding. The giants are probably stupid to the extreme due to that. Also, with only 100 giants at any time they will hardly be able to keep a culture or be educated.



                        Sinxe they hunt by smell, all the slayer needs to do is dressing scarecrows with clothes recently worn by actual people. Then fill the scarecrows with fresh but poisoned meat. Just wait for a few days before checking for each scarecrow.







                        share|improve this answer












                        share|improve this answer



                        share|improve this answer










                        answered Jan 8 at 10:50









                        RenanRenan

                        45.3k11104228




                        45.3k11104228












                        • $begingroup$
                          indeed they are stupid. They will be later controlled with the use of an artifact and this dude will fight them because the one controlling them want to get over a territory which includes his home place.
                          $endgroup$
                          – Edward Constantin
                          Jan 8 at 11:12


















                        • $begingroup$
                          indeed they are stupid. They will be later controlled with the use of an artifact and this dude will fight them because the one controlling them want to get over a territory which includes his home place.
                          $endgroup$
                          – Edward Constantin
                          Jan 8 at 11:12
















                        $begingroup$
                        indeed they are stupid. They will be later controlled with the use of an artifact and this dude will fight them because the one controlling them want to get over a territory which includes his home place.
                        $endgroup$
                        – Edward Constantin
                        Jan 8 at 11:12




                        $begingroup$
                        indeed they are stupid. They will be later controlled with the use of an artifact and this dude will fight them because the one controlling them want to get over a territory which includes his home place.
                        $endgroup$
                        – Edward Constantin
                        Jan 8 at 11:12











                        1












                        $begingroup$

                        In the interest of using your hero's power over magnetic fields, let's try a couple of other suggestions.




                        • Can he use the magnetic fields to accelerate a metal projectile beyond normal velocities? If you can take the average bow velocity of 300fps and get it up to 1200fps you could get away with something like "5 smooth ball-bearings and a sling".


                        • Can you manipulate a steel object at a distance? For instance, could you plant a knife somewhere in the giant's chest at range, then use magnetism to work the knife into his vitals?


                        • Now, to REALLY get rid of the giant, a magnetically confined Tokamak reactor...ok, off subject.


                        • As for armor, a magnetically controlled shield would be very entertaining. For balance, you would probably need to make the effort to your hero comparable to the force applied to it. (A giant would still knock it around)







                        share|improve this answer









                        $endgroup$













                        • $begingroup$
                          How do you think the magnetic shield would work? Can you come up with a design?
                          $endgroup$
                          – Edward Constantin
                          Jan 9 at 16:43






                        • 1




                          $begingroup$
                          Hmm, my imagination only got as far as taking a normal steel shield and levitating it. If we want to be more scientific, we could consider magnetic repulsion. Take a shield and put a large magnet on the back side. That becomes more like a magnetic shock absorber. All the energy of the impact goes into your character, but over an extended period of time. Maybe your character can transfer some of that to the planet's magnetic field to not take the full impact?
                          $endgroup$
                          – UrQuan3
                          Jan 10 at 18:33
















                        1












                        $begingroup$

                        In the interest of using your hero's power over magnetic fields, let's try a couple of other suggestions.




                        • Can he use the magnetic fields to accelerate a metal projectile beyond normal velocities? If you can take the average bow velocity of 300fps and get it up to 1200fps you could get away with something like "5 smooth ball-bearings and a sling".


                        • Can you manipulate a steel object at a distance? For instance, could you plant a knife somewhere in the giant's chest at range, then use magnetism to work the knife into his vitals?


                        • Now, to REALLY get rid of the giant, a magnetically confined Tokamak reactor...ok, off subject.


                        • As for armor, a magnetically controlled shield would be very entertaining. For balance, you would probably need to make the effort to your hero comparable to the force applied to it. (A giant would still knock it around)







                        share|improve this answer









                        $endgroup$













                        • $begingroup$
                          How do you think the magnetic shield would work? Can you come up with a design?
                          $endgroup$
                          – Edward Constantin
                          Jan 9 at 16:43






                        • 1




                          $begingroup$
                          Hmm, my imagination only got as far as taking a normal steel shield and levitating it. If we want to be more scientific, we could consider magnetic repulsion. Take a shield and put a large magnet on the back side. That becomes more like a magnetic shock absorber. All the energy of the impact goes into your character, but over an extended period of time. Maybe your character can transfer some of that to the planet's magnetic field to not take the full impact?
                          $endgroup$
                          – UrQuan3
                          Jan 10 at 18:33














                        1












                        1








                        1





                        $begingroup$

                        In the interest of using your hero's power over magnetic fields, let's try a couple of other suggestions.




                        • Can he use the magnetic fields to accelerate a metal projectile beyond normal velocities? If you can take the average bow velocity of 300fps and get it up to 1200fps you could get away with something like "5 smooth ball-bearings and a sling".


                        • Can you manipulate a steel object at a distance? For instance, could you plant a knife somewhere in the giant's chest at range, then use magnetism to work the knife into his vitals?


                        • Now, to REALLY get rid of the giant, a magnetically confined Tokamak reactor...ok, off subject.


                        • As for armor, a magnetically controlled shield would be very entertaining. For balance, you would probably need to make the effort to your hero comparable to the force applied to it. (A giant would still knock it around)







                        share|improve this answer









                        $endgroup$



                        In the interest of using your hero's power over magnetic fields, let's try a couple of other suggestions.




                        • Can he use the magnetic fields to accelerate a metal projectile beyond normal velocities? If you can take the average bow velocity of 300fps and get it up to 1200fps you could get away with something like "5 smooth ball-bearings and a sling".


                        • Can you manipulate a steel object at a distance? For instance, could you plant a knife somewhere in the giant's chest at range, then use magnetism to work the knife into his vitals?


                        • Now, to REALLY get rid of the giant, a magnetically confined Tokamak reactor...ok, off subject.


                        • As for armor, a magnetically controlled shield would be very entertaining. For balance, you would probably need to make the effort to your hero comparable to the force applied to it. (A giant would still knock it around)








                        share|improve this answer












                        share|improve this answer



                        share|improve this answer










                        answered Jan 8 at 17:35









                        UrQuan3UrQuan3

                        1891




                        1891












                        • $begingroup$
                          How do you think the magnetic shield would work? Can you come up with a design?
                          $endgroup$
                          – Edward Constantin
                          Jan 9 at 16:43






                        • 1




                          $begingroup$
                          Hmm, my imagination only got as far as taking a normal steel shield and levitating it. If we want to be more scientific, we could consider magnetic repulsion. Take a shield and put a large magnet on the back side. That becomes more like a magnetic shock absorber. All the energy of the impact goes into your character, but over an extended period of time. Maybe your character can transfer some of that to the planet's magnetic field to not take the full impact?
                          $endgroup$
                          – UrQuan3
                          Jan 10 at 18:33


















                        • $begingroup$
                          How do you think the magnetic shield would work? Can you come up with a design?
                          $endgroup$
                          – Edward Constantin
                          Jan 9 at 16:43






                        • 1




                          $begingroup$
                          Hmm, my imagination only got as far as taking a normal steel shield and levitating it. If we want to be more scientific, we could consider magnetic repulsion. Take a shield and put a large magnet on the back side. That becomes more like a magnetic shock absorber. All the energy of the impact goes into your character, but over an extended period of time. Maybe your character can transfer some of that to the planet's magnetic field to not take the full impact?
                          $endgroup$
                          – UrQuan3
                          Jan 10 at 18:33
















                        $begingroup$
                        How do you think the magnetic shield would work? Can you come up with a design?
                        $endgroup$
                        – Edward Constantin
                        Jan 9 at 16:43




                        $begingroup$
                        How do you think the magnetic shield would work? Can you come up with a design?
                        $endgroup$
                        – Edward Constantin
                        Jan 9 at 16:43




                        1




                        1




                        $begingroup$
                        Hmm, my imagination only got as far as taking a normal steel shield and levitating it. If we want to be more scientific, we could consider magnetic repulsion. Take a shield and put a large magnet on the back side. That becomes more like a magnetic shock absorber. All the energy of the impact goes into your character, but over an extended period of time. Maybe your character can transfer some of that to the planet's magnetic field to not take the full impact?
                        $endgroup$
                        – UrQuan3
                        Jan 10 at 18:33




                        $begingroup$
                        Hmm, my imagination only got as far as taking a normal steel shield and levitating it. If we want to be more scientific, we could consider magnetic repulsion. Take a shield and put a large magnet on the back side. That becomes more like a magnetic shock absorber. All the energy of the impact goes into your character, but over an extended period of time. Maybe your character can transfer some of that to the planet's magnetic field to not take the full impact?
                        $endgroup$
                        – UrQuan3
                        Jan 10 at 18:33











                        1












                        $begingroup$

                        Your main character could use a rope or net studded or made of metal. The rope wraps around the giant to bring them down, and your main character can control the movement of the rope/net very precisely by manipulating the metal in it.
                        Once bound, you have a number of ways to kill it; personally I would recommend just stabbing it until it bleeds out. Go for the eyes first, in case your character’s rope inexplicably fails.



                        For armor, I agree with others; the strength of the armor doesn't matter when going against giants, so ease of movement is key. Clothing or leather is best.






                        share|improve this answer









                        $endgroup$


















                          1












                          $begingroup$

                          Your main character could use a rope or net studded or made of metal. The rope wraps around the giant to bring them down, and your main character can control the movement of the rope/net very precisely by manipulating the metal in it.
                          Once bound, you have a number of ways to kill it; personally I would recommend just stabbing it until it bleeds out. Go for the eyes first, in case your character’s rope inexplicably fails.



                          For armor, I agree with others; the strength of the armor doesn't matter when going against giants, so ease of movement is key. Clothing or leather is best.






                          share|improve this answer









                          $endgroup$
















                            1












                            1








                            1





                            $begingroup$

                            Your main character could use a rope or net studded or made of metal. The rope wraps around the giant to bring them down, and your main character can control the movement of the rope/net very precisely by manipulating the metal in it.
                            Once bound, you have a number of ways to kill it; personally I would recommend just stabbing it until it bleeds out. Go for the eyes first, in case your character’s rope inexplicably fails.



                            For armor, I agree with others; the strength of the armor doesn't matter when going against giants, so ease of movement is key. Clothing or leather is best.






                            share|improve this answer









                            $endgroup$



                            Your main character could use a rope or net studded or made of metal. The rope wraps around the giant to bring them down, and your main character can control the movement of the rope/net very precisely by manipulating the metal in it.
                            Once bound, you have a number of ways to kill it; personally I would recommend just stabbing it until it bleeds out. Go for the eyes first, in case your character’s rope inexplicably fails.



                            For armor, I agree with others; the strength of the armor doesn't matter when going against giants, so ease of movement is key. Clothing or leather is best.







                            share|improve this answer












                            share|improve this answer



                            share|improve this answer










                            answered Jan 9 at 22:14









                            ParalyzoidParalyzoid

                            211




                            211























                                1












                                $begingroup$

                                How strong and precise magnetic field are we talking about here?



                                It strong enough, he can give cannon balls so much kinetic energy (preferably from distance, for example in high levitating matal chair with seatbelts so no giant can reach him but he has nice view of battlefield) to massacre the poor giants to bloody equivalent of mashed potatoes.



                                Even worse, if can generate ridiculously strong magnetic field (like magnetars) he would be God-like, being able to distort atoms themselves in Giant bodies (and elsewhere).






                                share|improve this answer











                                $endgroup$













                                • $begingroup$
                                  He is not that strong. He can move heavy objects but not anything like creating magnetars
                                  $endgroup$
                                  – Edward Constantin
                                  Jan 9 at 16:42
















                                1












                                $begingroup$

                                How strong and precise magnetic field are we talking about here?



                                It strong enough, he can give cannon balls so much kinetic energy (preferably from distance, for example in high levitating matal chair with seatbelts so no giant can reach him but he has nice view of battlefield) to massacre the poor giants to bloody equivalent of mashed potatoes.



                                Even worse, if can generate ridiculously strong magnetic field (like magnetars) he would be God-like, being able to distort atoms themselves in Giant bodies (and elsewhere).






                                share|improve this answer











                                $endgroup$













                                • $begingroup$
                                  He is not that strong. He can move heavy objects but not anything like creating magnetars
                                  $endgroup$
                                  – Edward Constantin
                                  Jan 9 at 16:42














                                1












                                1








                                1





                                $begingroup$

                                How strong and precise magnetic field are we talking about here?



                                It strong enough, he can give cannon balls so much kinetic energy (preferably from distance, for example in high levitating matal chair with seatbelts so no giant can reach him but he has nice view of battlefield) to massacre the poor giants to bloody equivalent of mashed potatoes.



                                Even worse, if can generate ridiculously strong magnetic field (like magnetars) he would be God-like, being able to distort atoms themselves in Giant bodies (and elsewhere).






                                share|improve this answer











                                $endgroup$



                                How strong and precise magnetic field are we talking about here?



                                It strong enough, he can give cannon balls so much kinetic energy (preferably from distance, for example in high levitating matal chair with seatbelts so no giant can reach him but he has nice view of battlefield) to massacre the poor giants to bloody equivalent of mashed potatoes.



                                Even worse, if can generate ridiculously strong magnetic field (like magnetars) he would be God-like, being able to distort atoms themselves in Giant bodies (and elsewhere).







                                share|improve this answer














                                share|improve this answer



                                share|improve this answer








                                edited Jan 10 at 20:34

























                                answered Jan 8 at 17:32









                                Matija NalisMatija Nalis

                                48626




                                48626












                                • $begingroup$
                                  He is not that strong. He can move heavy objects but not anything like creating magnetars
                                  $endgroup$
                                  – Edward Constantin
                                  Jan 9 at 16:42


















                                • $begingroup$
                                  He is not that strong. He can move heavy objects but not anything like creating magnetars
                                  $endgroup$
                                  – Edward Constantin
                                  Jan 9 at 16:42
















                                $begingroup$
                                He is not that strong. He can move heavy objects but not anything like creating magnetars
                                $endgroup$
                                – Edward Constantin
                                Jan 9 at 16:42




                                $begingroup$
                                He is not that strong. He can move heavy objects but not anything like creating magnetars
                                $endgroup$
                                – Edward Constantin
                                Jan 9 at 16:42





                                protected by L.Dutch Jan 8 at 10:24



                                Thank you for your interest in this question.
                                Because it has attracted low-quality or spam answers that had to be removed, posting an answer now requires 10 reputation on this site (the association bonus does not count).



                                Would you like to answer one of these unanswered questions instead?



                                Popular posts from this blog

                                Mario Kart Wii

                                What does “Dominus providebit” mean?

                                Antonio Litta Visconti Arese